www.vorhilfe.de
- Förderverein -
Der Förderverein.

Gemeinnütziger Verein zur Finanzierung des Projekts Vorhilfe.de.
Hallo Gast!einloggen | registrieren ]
Startseite · Mitglieder · Impressum
Forenbaum
^ Forenbaum
Status VH e.V.
  Status Vereinsforum

Gezeigt werden alle Foren bis zur Tiefe 2

Navigation
 Startseite...
 Suchen
 Impressum
Das Projekt
Server und Internetanbindung werden durch Spenden finanziert.
Organisiert wird das Projekt von unserem Koordinatorenteam.
Hunderte Mitglieder helfen ehrenamtlich in unseren moderierten Foren.
Anbieter der Seite ist der gemeinnützige Verein "Vorhilfe.de e.V.".
Partnerseiten
Weitere Fächer:

Open Source FunktionenplotterFunkyPlot: Kostenloser und quelloffener Funktionenplotter für Linux und andere Betriebssysteme
Forum "Reelle Analysis mehrerer Veränderlichen" - Maximumsnorm als Grenzfall
Maximumsnorm als Grenzfall < mehrere Veränderl. < reell < Analysis < Hochschule < Mathe < Vorhilfe
Ansicht: [ geschachtelt ] | ^ Forum "Reelle Analysis mehrerer Veränderlichen"  | ^^ Alle Foren  | ^ Forenbaum  | Materialien

Maximumsnorm als Grenzfall: Frage (beantwortet)
Status: (Frage) beantwortet Status 
Datum: 14:44 Sa 30.11.2019
Autor: Boogie2015

Eingabefehler: "\left" und "\right" müssen immer paarweise auftreten, es wurde aber ein Teil ohne Entsprechung gefunden (siehe rote Markierung)

Guten Vormittag, ich möchte gerne wissen, warum $\lim\limits_{ p \rightarrow \infty} \vert \vert x \vert \vert_{p} = \lim\limits_{ p \rightarrow \infty}\left (  \sum\limits_{j = 1}^{d} \vert x_{j} \vert ^{p} \right)^{\frac{1}{p}} = max_{j = 1, \ldos, d} \vert x_{j} \vert = \vert \vert x \vert \vert_{\infty}$ gilt.


Ich habe mir dazu den Beweis auf Wikipedia angeschaut, also  diesen:

________________________________________________________________________________________________________________________________

$\lim _{p\rightarrow \infty }\left(\sum _{i=1}^{n}|x_{i}|^{p}\right)^{1/p}\!\!\!\!\!=\|x\|_{\infty }\cdot \lim _{p\rightarrow \infty }\left(\sum _{i=1}^{n}\left({\frac {|x_{i}|}{\|x\|_{\infty }}}\right)^{p}\right)^{1/p}\!\!\!\!\!=\|x\|_{\infty }\cdot \lim _{p\rightarrow \infty }S^{1/p}=\|x\|_{\infty }}$,

da für die Summe $1 \leq S \leq n$ gilt und somit der Grenzwert von $ \sqrt[p]{S}$ für $ p\rightarrow \infty $ gleich Eins ist. Die untere Schranke von $S$ wird dabei für einen Vektor angenommen, dessen Komponenten bis auf eine alle gleich Null sind, und die obere Schranke$n$ für einen Vektor, dessen Komponenten alle den gleichen Betrag besitzen.  Durch Weglassen des Limes ist so auch ersichtlich, dass die Maximumsnorm niemals größer als die $p$ -Normen ist.


________________________________________________________________________________________________________________________________

Den Beweis dazu habe ich eigentlich ganz gut verstanden, aber der ist noch nicht so richtig intuitiv, weil da mit $\frac{\vert \vert x \vert \vert_{\infty}}{\vert \vert x \vert \vert_{\infty}}$ gespielt wird.

Ich meine, die künstliche $1$ fügt man nur hinzu, weil man am Ende eh weiß, dass der Grenzwert eben die Maximumsnorm ist. Aber das wusste man ja am Anfang nicht. Daher hätten sie nicht auf so einem Ansatz kommen können.


Meine Frage ist: Wie komme ich auf den selben Grenzwert, ohne mitten in der Rechnung das Maximum aller Beträge zu erhalten oder eine derartige künstliche $1$ hinzuzufügen?


Ich hoffe, ihr wisst, was ich meine.


Wie schaffe ich es also auf natürlichem Weg, die Gleichung $\lim _{p\rightarrow \infty }\left(\sum _{i=1}^{n}|x_{i}|^{p}\right)^{1/p}\!\!\!\!\! = \|x\|_{\infty }}$ zu zeigen?




Ich würde z.B. so anfangen:


$\lim _{p\rightarrow \infty }\left(\sum _{i=1}^{n}|x_{i}|^{p}\right)^{1/p}\!\!\!\!\! = \lim _{p\rightarrow \infty }  \sqrt[p]{\vert x_{1} \vert^{p} + \vert x_{2} \vert^{p} + \vert x_{3} \vert^{p} +  \ldots + \vert x_{d} \vert^{p}} =  \lim _{p\rightarrow \infty }  \sqrt[p]{\vert x_{1} \vert^{p} \left 1 +\frac{\vert x_{2} \vert^{p}}{\vert x_{1} \vert^{p}} + \frac{\vert x_{3} \vert^{p}}{\vert x_{1} \vert^{p}} + \ldots + \frac{\vert x_{d} \vert^{p}}{\vert x_{1} \vert^{p}} \right )} = \lim _{p\rightarrow \infty }  \sqrt[p]{\vert x_{1} \vert^{p} \left ( 1 + \left ( \frac{\vert x_{2} \vert }{\vert x_{1} \vert} \right )^{p} + \left ( \frac{\vert x_{3} \vert }{\vert x_{1} \vert} \right )^{p} + \ldots + \left ( \frac{\vert x_{d} \vert }{\vert x_{1} \vert} \right )^{p} \right )}$


$ = \lim _{p\rightarrow \infty }  \vert x_{1} \vert \cdot   \sqrt[p]{ \left ( 1 + \left ( \frac{\vert x_{2} \vert }{\vert x_{1} \vert} \right )^{p} + \left ( \frac{\vert x_{3} \vert }{\vert x_{1} \vert} \right )^{p} + \ldots + \left ( \frac{\vert x_{d} \vert }{\vert x_{1} \vert} \right )^{p} \right )}$

Und so weiter. Aber das wird eine ewige Rechnung. Gibt es andere Methoden?





lg, boogie

        
Bezug
Maximumsnorm als Grenzfall: Antwort
Status: (Antwort) fertig Status 
Datum: 15:11 Sa 30.11.2019
Autor: fred97

Was mit S gemeint ist, hast du nicht  gesagt.

Ich beweise obige Grenzwertbeziehung so, dabei  genügt es, den  Fall d=2  zu betrachten ( die Idee für den allgemeinen Fall dürfte dann klar sein.)

Seien  a und b nichtnegative Zahlen und a [mm] \le [/mm] b. Es folgt

$b [mm] \le (b^{p}+a^{p})^{1/p} \le (2b^{p})^{1/p}=2^{1/p}b. [/mm] $

mit $p [mm] \to \infty [/mm]  $ folgt das Resultat.

Bezug
Ansicht: [ geschachtelt ] | ^ Forum "Reelle Analysis mehrerer Veränderlichen"  | ^^ Alle Foren  | ^ Forenbaum  | Materialien


^ Seitenanfang ^
ev.vorhilfe.de
[ Startseite | Mitglieder | Impressum ]